Questions tagged [arithmetic-functions]

The tag has no usage guidance.

Filter by
Sorted by
Tagged with
1 vote
0 answers
95 views

Relationship between two types of partition functions

Referring to this unanswered question on MS, I'm posting the same question here: For $s\in \mathbb{C},\Re(s)>1 $, consider: $$\prod_{k=1}^{\infty}\prod_{n=2}^{\infty}\frac{1}{1-n^{-ks}}= \prod_{k=1}...
mohammad-83's user avatar
3 votes
2 answers
451 views

Approximation of partial sum over prime omega function

I asked the question in Math StackExchange. Link: https://math.stackexchange.com/questions/4765476/approximation-of-partial-sum-over-prime-omega-function I haven't got any response yet. Here are the ...
piepie's user avatar
  • 221
1 vote
0 answers
138 views

On two formulas involving the $k$-fold divisor function $d_k$ and the function $r_k$

I have a puzzle which needs some help form the experts here. Let $d(n)$ be the divisor function, and $d_k(n)$ the $k$-fold divisor function. I) It is known that, for any positive integer $h$, $$d(n+h)...
hofnumber's user avatar
  • 553
7 votes
1 answer
189 views

Are there infinite numbers of the form $\sigma_1(n)=\sigma_1(m)=p$, or is there only one?

I put forward a hypothesis in number theory, it is as follows.$ \sigma_1(n)=\sigma_1(m)=p$, where $\sigma_1$ is the divisor sum function, $n,m\in \mathbb N$, and $p$ is prime. I recently noticed and ...
Arsen Vardanyan's user avatar
0 votes
1 answer
263 views

Estimating a sum involving the von Mangoldt function

I'd like to know the estimate of the following sum $$\sum_{n\leq x}\sum_{d|n}\Lambda(d)\frac{\phi(d)}{d} $$ where $\Lambda(d)$ is the von mangoldt function and $\phi(d)$ is the Euler totient function. ...
Beta's user avatar
  • 365
4 votes
0 answers
76 views

On Carmichael function and aliquot parts of odd perfect numbers

I've asked nine months ago this question on Mathematics Stack Exchange with identifier 4430381 and same title. There is not answer for this question on Mathematics Stack Exchange, I wondered if this ...
user142929's user avatar
0 votes
0 answers
64 views

Around similar inequalities than an inequality due to Nicolas, that involve products of consecutive Ramanujan primes

This is cross-posted (and this post is a version to ask just around the veracity of Conjecture 1) as the post with identifier 3594907 and same title), that I've edited on Mathematics Stack Exchange ...
user142929's user avatar
1 vote
1 answer
335 views

On equations with arithmetic functions [closed]

Is this good topic for research: equations with arithmetic functions, for example equations like $\varphi(n)=\sigma(n)$ or $\varphi(n)+\sigma(n)=d(n)$ ? If Anyone here have an advise please tell me ...
Omega's user avatar
  • 31
-9 votes
1 answer
494 views

Arithmetic billiards, prime numbers and the Goldbach conjecture

I've edited the following post on Mathematics Stack Exchange, (now closed, at that date I'm suspended) with identifier 4510963, please let me to know if you've some doubt or I can improve the post. On ...
user142929's user avatar
6 votes
1 answer
203 views

Upper bound on minimum number of prime factors in short intervals

Suppose that $H = H(X)$ is some quantity growing with $X$. Are there any bounds on $$F(X, H) = \min_{X < n\le X + H} \omega(n)?$$ It isn't hard to obtain a lower bound $\max_{x\sim X} F(X, H)\gg \...
Mayank Pandey's user avatar
0 votes
0 answers
86 views

A similar inequality for the Dedekind psi function, than an inequality stated by Schinzel

I would like to ask about the next question that seems to me interesting. I know an article that was written by Andrzej Schinzel in which he stated Lemma 2. In this post we denote the Dedekind psi ...
user142929's user avatar
1 vote
1 answer
81 views

On the behaviour for the quotient involving Fermat numbers of $\frac{\psi(F_m)}{F_m}$ where $\psi(x)$ denotes the Dedekind psi function

In this post we denote the Dedekind psi function as $\psi(m)$ for integers $m\geq 1$. This is an important arithmetic fuction in several subjects of mathematics. As reference I add the Wikipedia ...
user142929's user avatar
2 votes
1 answer
326 views

A conjecture concerning the equation $\sigma\left(\square\right)=\text{prime}$

I can deduce the following simple proposition, the definitions for $\sigma(x)$ the sum of divisors functions and $\varphi(x)$ the Euler totient function are assumed. After I present a conjecture that ...
user142929's user avatar
1 vote
1 answer
105 views

Periodic sequences of integers generated by $a_{n+1}=\frac{\operatorname{rad}(pa_{n})}{p}+\frac{\operatorname{rad}(qa_{n-1})}{q}$

Let's define the radical of the positive integer $n$ as $$\operatorname{rad}(n)=\prod_{\substack{p\mid n\\ p\text{ prime}}}p$$ and consider the sequence $$a_{n+1}=\frac{\operatorname{rad}(p\cdot a_{n})...
Augusto Santi's user avatar
1 vote
0 answers
143 views

A definition related to pseudoprimes and the Dedekind psi function

In this post we consider that $\psi(k)$ denotes the Dedekind psi function. Wikipedia has an artcle dedicated to this arithmetic function Dedekind psi function defined for a positive integers $m>1$ ...
user142929's user avatar
4 votes
0 answers
228 views

On the density of a particular subset of integers

Given a positive integer $n$ in the standard form $$n=\prod_k p_k^{\alpha_k}$$ and the arithmetic function (investigated by Erdős in this paper) $$A(n)=\sum_k \alpha_k p_k$$ let's define the subset $E$...
Augusto Santi's user avatar
18 votes
1 answer
584 views

For which $n$ is $\sum_{k=1}^n 1 / \varphi(k)$ an integer?

For which positive integers $n$ is the sum $\sum_{k=1}^n 1 / \varphi(k)$ an integer? Here $\varphi$ is the Euler totient function. The question is a "totient-analog" of the well-known result ...
annie's user avatar
  • 453
0 votes
0 answers
37 views

Number of different factorizations

I define $\nu(n)$ the number of different factorizations for an integer $n$. I know there are papers about $\delta(n)$ the number of dividers for an integer $n$ (Landau, Euler, Dirichlet) but I still ...
Aileann D. PRET's user avatar
3 votes
2 answers
229 views

Help with R. Ryan's "A simpler dense proof regarding the abundancy Index."

I'm reading Richard Ryan's article "A simpler dense proof regarding the abundancy index" and got stuck in his proof for Theorem 2. The Theorem is stated as follows: Suppose we have a ...
jvkloc's user avatar
  • 133
0 votes
0 answers
128 views

How to estimate sums over arithmetic progressions?

For $x>1$ $$ N(x)=\sum_{0<n<x \\n \equiv 1 \pmod 4\\ n\text{ squarefree}} 1 $$ How to estimate $N(x)$'s order? (Like $N(x) \sim Ax$) Furthermore, for $n=p_1p_2\cdots p_v$, define $\alpha (n)=...
five's user avatar
  • 1
6 votes
1 answer
371 views

Arithmetic properties of positively reduced $2\times 2$-matrices

Call a $2\times 2$ matrix with coefficients in $\{0,1,2,3,\ldots\}$ positively reduced if any row or column reduction (given by replacing a row/column by itself minus the other row/column) produces at ...
Roland Bacher's user avatar
4 votes
0 answers
79 views

Joint mean values of arithmetic functions in sequences and families of sequences

This is a bit of a follow up question to this question I asked a couple days ago. The main content of that post can be phrased as asking for a nontrivial lower bound on the sum $$ \sum_{n\leq x} \...
Joshua Stucky's user avatar
0 votes
1 answer
216 views

Correlating the von Mangoldt function with periodic sequences

The Dirichlet inverse of the Euler totient function is: $$\varphi^{-1}(n) = \sum_{d \mid n} \mu(d)d \tag{1}$$ and the von Mangoldt function can be expanded/computed as: $$\Lambda(n) = \sum\limits_{k=1}...
Mats Granvik's user avatar
  • 1,133
3 votes
1 answer
132 views

Is it possible to find an estimate of $\sum_{k=1}^n\frac1{\varphi(k\cdot p_k)}$?

Is it possible to find an estimate of the summation $$s(n)=\sum_{k=1}^n\frac1{\varphi(k\cdot p_k)}$$ where $\varphi(n)$ is the totient function and $p_k$ the k-th prime? The corresponding series seems ...
Augusto Santi's user avatar
11 votes
2 answers
875 views

Have any proposals been advanced for the analytic continuation of the divisor function?

While I was working on the evaluation of a certain series, the following limit came up: \begin{align} \lim_{n \to 1} \frac{d(n)-1}{n(n-1)} &= \lim_{n \to 1} \frac{d'(n)}{2n-1} \\ &= d'(1) .\...
Max Muller's user avatar
  • 4,435
2 votes
0 answers
693 views

Polynomials for the indicator function

The (one-variable) indicator function (or characteristic function) is defined as $f_{t^*}:\mathbb{Z}_q\to \mathbb{Z}_q$ satisfying that $f_{t^*}(t)=1$ if $t=t^*$ and $f_{t^*}(t)=0$, otherwise. (Here $...
Huy Le's user avatar
  • 31
7 votes
2 answers
433 views

Prove that two functions are equal only when $s \equiv \pm r^{\pm 1} \pmod{q}$

Let us fix a positive integer $q$, and let us define a functions $P: \mathbb{Z}\times \mathbb{N} \to \mathbb{Z}$ as follows: $$ P(s,t) := \sum_{j=1}^t \left\lfloor \frac{j (s-1) + t}{q} \right\rfloor$$...
Luis Ferroni's user avatar
  • 1,879
2 votes
0 answers
327 views

Mertens Bound and the Riemann Hypothesis

Let $M(x)$ denote the Mertens function ($M(x)=\sum_{i=1}^{x}\mu(i)$ where $\mu(i)$ is the Möbius function) and let $\Lambda(i)$ denote the Mangoldt function ($\Lambda(i)$ equals $\log(p)$ if $i=p^{m}$ ...
Sourangshu Ghosh's user avatar
2 votes
0 answers
129 views

What is known about absolute convergence of Dirichlet inverses?

Given an arithmetic function $f$ such that the partial sums $\sum_{n \leq x} |f(n)|$ converge as $x$ approaches $\infty$, are there any results concerning the convergence properties of the series of ...
Jack Ceroni's user avatar
2 votes
0 answers
64 views

The number of elements with order less than $k$ in a larger cyclic group

I am working on a problem where it has become important to count (or at least bound from above and below) the number of elements of ${\bf Z}/n{\bf Z}$ that have order less than a given $k$, where $2\...
Marcel K. Goh's user avatar
5 votes
0 answers
253 views

Are the numbers $\varphi(n^2)\sigma(n^2)\ (n=1,2,3,\ldots)$ pairwise distinct?

Let $\varphi$ be Euler's totient function, and let $\sigma(n)=\sum_{d\mid n}d$ for $n=1,2,3,\ldots$. Both $\varphi$ and $\sigma$ are multiplicative functions. It is easy to see that the numbers $$\...
Zhi-Wei Sun's user avatar
  • 14.4k
1 vote
1 answer
219 views

Sign changes of a sequence

Let $f$ be an arithmetical function. Suppose that $f(n)>0$ if $n$ is in an integer set $A$ and that $f(n)<0$ for another integer set $B.$ Is there a result from number theory or an elementary ...
Khadija Mbarki's user avatar
0 votes
1 answer
353 views

Where can I find the problem by Lagarias?

Jeffrey Lagarias proved, unconditionally, that: $$ \sigma(n)<H_n+2\exp(H_n)\log(H_n)\qquad n>1 $$ This was posed as a problem in: J. C. Lagarias, Problem 10949: A generous bound for divisor ...
The Company's user avatar
2 votes
1 answer
613 views

$\frac{\sigma(n)}{n} < e \ln \ln (n)$ is true?

In Guy Robin, Grandes valeurs de la fonction somme des diviseurs et hypothèse de Riemann, J. Math. Pures Appl. 63 (1984), 187–213 (pdf) we find the following result: If the Riemann hypothesis is true ...
The Company's user avatar
3 votes
1 answer
150 views

Arithmetical function comparable to sine function [closed]

I was wondering if there exists or can we construct (using known arithmetic functions) an arithmetical function that has the same behaviour of the function sine or comparable to it (I mean that ...
Khadija Mbarki's user avatar
4 votes
1 answer
278 views

Generalization of the The Liouville Lambda function

Let $n=p^{\alpha_1}_1 \cdots p^{\alpha_m}_m,$ and define $$\lambda_k(n)= (-1)^{ [\frac{\Omega(n)}{k} ]},$$ where $\Omega(n)= \alpha_1 + \cdots + \alpha_k,$ and $[\cdot]$ is the floor function. For $...
Farzad Aryan's user avatar
6 votes
0 answers
500 views

Does the equation $\sigma(\sigma(x^2))=2x\sigma(x)$ have any odd solutions?

This question was posted in MSE in early August 2020. It did garner several upvotes, but did not receive any responses. I have therefore cross-posted it here, hoping that it gets answered. Let $\...
Jose Arnaldo Bebita Dris's user avatar
1 vote
0 answers
170 views

Is $T(n)=\sum_{k=1}^{n}\frac{\lambda(k)\Lambda(k)}{k} \geq 0$ and what is the upper bound of $T(x)=\sum_{n\leq x} \lambda(n)\Lambda(n) $?

Let $\Lambda(n)$ denote the von Mangoldt function: $\Lambda(n)=\log p$ when $n=p^e$ is a prime power ($e\ge 1$) and $\Lambda(n)=0$ otherwise. and $\lambda(n)$ be Liouville Function, , I'm interested ...
zeraoulia rafik's user avatar
0 votes
0 answers
91 views

Arithmetic expansion of harmonic sum

Note: I have modified the initial question as follows: Let $w_1, w_2, \ldots, w_d$ be positive weights, and $x_1, x_2, \ldots, x_d$ be positive variables. Now, let us consider the following harmonic ...
Yoshitaka's user avatar
5 votes
3 answers
785 views

Is the number of solutions of $\phi(x)=n!$ bounded? If yes, what is its bound?

Pillai showed in 1929 that the function $A(n)$ giving the number solutions of the equation $\phi(x)=n$ is unbounded in (S. Pillai, On some functions connected with $\varphi(n)$, Bull. Amer. Math. Soc. ...
zeraoulia rafik's user avatar
8 votes
1 answer
316 views

On the density map of the abundancy index

Let $σ$ be the sum-of-divisors function. Let $σ(n)/n$ be the abundancy index of $n$. Consider the density map $$f(x) = \lim_{N \to \infty} f_N(x) \ \ \text{ with } \ \ f_N(x) = \frac{1}{N} \#\{ 1 \...
Sebastien Palcoux's user avatar
2 votes
0 answers
108 views

On variations of a claim due to Kaneko in terms of Lehmer means

This post is cross posted from Mathematics Stack Exchange, due that there was a mistake from my part (see the excellent partial answer and my thread of edits of my question on MSE) this post on ...
user142929's user avatar
2 votes
0 answers
71 views

Write large $n$ as $n_1+\ldots+n_k\ (n_1<\ldots<n_k)$ with $\varphi(n_1),\ldots,\varphi(n_k)\in\{x^k:\ x\in\mathbb Z\}$

Let $\varphi$ denote Euler's totient function. QUESTION. Is it true that for each positive integer $k$ large integers $n$ can be written as $n_1+\ldots+n_k$ with $n_1,\ldots,n_k$ distinct positive ...
Zhi-Wei Sun's user avatar
  • 14.4k
0 votes
0 answers
148 views

On the equation that involves the Dedekind psi function $\psi(x)=n$ with unique solution $x$, for a fixed integer $n\geq 1$

The Dedekind psi function is defined for a positive integer $m>1$ as $$\psi(m)=m\prod_{\substack{p\mid m\\p\text{ prime}}}\left(1+\frac{1}{p}\right)\tag{1}$$ with the definition $\psi(1)=1$. See ...
user142929's user avatar
0 votes
2 answers
348 views

What is the dirichlet series of $f(n)=\sum_{d | n}(\log d) / d$ function? [closed]

My opinion is ; We may use id(d)=d arithmetic function and log*id dirichlet convolution in the question. i thought that ; when we multiply and divide n with $(\log d) / d$ we obtain $F(S)=\sum_{n=...
user1062's user avatar
  • 105
0 votes
0 answers
179 views

When is $\phi(a^n+b^n+c^n)=0\mod n$?

A corollary Zsigmondy's Theorem leads to the following congruence (one can look to $(24)$),$\phi(a^n+b^n)=0\mod n$ whenever $a, b$ are coprime and $n \neq 2$ and $(a,b)\neq(1,1)$. (Here $\phi$ is the ...
user avatar
1 vote
1 answer
188 views

Bounds for two arithmetic functions, when one assumes that $n$ are odd perfect numbers

For an integer $n>1$ in this post we denote the Dedekind psi function as $\psi(n)=n\prod_{\substack{p\mid n\\p\text{ prime}}}\left(1+\frac{1}{p}\right)$ and the product of distinct primes dividing ...
user142929's user avatar
2 votes
1 answer
344 views

Is there a smallest $r$ such that $n+\varphi(n)=\displaystyle \prod_{i=1}^r q_i$ always has solutions for mutually different odd primes $q_i $?

While discussing with Peter in one of the chatrooms on MSE I proposed an idea to try to find smallest natural number $r$ such that $n+\varphi(n)=\displaystyle \prod_{i=1}^r q_i$ has solutions for ...
user avatar
0 votes
0 answers
90 views

Set of primes $p_{1}\equiv 3 \bmod p_{2}$ such that $\phi(2^{\frac{{p_1}-3}{p_2}}-1)\equiv 0 \bmod p_1$ with $p_1,p_2\equiv 3\bmod 4$?

let $p_1$ and $p_2$ be positive primes such that $p_1,p_2 \equiv 3\bmod 4$ and $\phi$ is the Euler totiont function , I want to find the Set of primes $p_{1}\equiv 3 \bmod p_{2}$ such that $\phi(2^...
zeraoulia rafik's user avatar
2 votes
0 answers
232 views

If some numbers satisfy this divisibility condition with $\sigma$ and $\varphi$, are they necessarily multiples of $6$?

After doing some computations of the divisibility of $\sigma(n)$ by $n+ \varphi(n)$, mostly with Peter´s help, we found these solutions: $n=2, 456, 828, 7584 ,33462 , 1357440, 1596048 ,1964544 ,...
user avatar